Step 2 CK (CCSSA) Form 6 - Step Prep

You might also like

Download as pdf or txt
Download as pdf or txt
You are on page 1of 56

STEP PREP

Helping medical students navigate 3rd year exams

MARCH 8, 2020 BY STEPPREP12345

Step 2 CK (CCSSA) Form 6


Exam Section 1:

1. A 14-month-old girl is brought to the physician because of a 14-hour history…

Water-soluble contrast enema

Intussusception often presents in a child 6-36 months old with abdominal pain relieved by
drawing knees to the chest +/- bloody diarrhea +/- vomiting. In setting of suspected
intussusception, next best step is a contrast/air enema which is both therapeutic and
diagnostic.

X-ray of upper GI tract with contrast indicated in setting of suspected malrotation with
volvulus (newborn with abdominal pain + bilious vomiting + distention)
Corticosteroid enemas sometimes used in ulcerative colitis

2. A 52-year old woman comes to the physician because…

Decreased androgens

Ovaries not only produce estrogens, but also produce androgens that play a central role in
libido (among other things)

3. A 27-year old man comes to the physician because of a 1-week history…

Diffuse hypokinesia and dilation of the ventricles

Young patient who presents with acute signs of heart failure (elevated JVP, bilateral
basilar crackles, displaced PMI, S3, lower extremity edema, etc.) most likely represents
myocarditis, especially in setting of recent viral URI
Asymmetric septal hypertrophy = HOCM = Sudden cardiac death or exertional syncope in
young adult
Bicuspid aortic valve with stenosis presents as symptomatic AS (syncope, angina,
dyspnea) often in a 50-60 year old

4. During the past 4 months, a 32-year-old woman has had pain…

Sjogren syndrome

Mucosal dryness in young or middle-aged woman, especially in one with other


autoimmune disease such as this patient (T1DM), should make you think of Sjogren
syndrome
Important to contrast with age-related sicca syndrome, which is present in ~25% of
individuals over 65 years old and leads to sensation of dry eyes and/or dry mouth with a
negative ANA (if they are feeling generous and give you this information)

5. A 24-year-old primigravid woman at 18 weeks’ gestation…

In ammatory bowel disease

IBD should always be suspected in a young adult without a clear trigger (international
travel, etc.) who presents with increased bowel movements, especially if stools are
covered with blood and/or patient has associated abdominal pain
Diseases associated with erythema nodosum = IBD, tuberculosis, systemic fungal
infections (cocci, etc.), sarcoidosis, Behcet’s disease, strep pharyngitis (most common
trigger)
Important to distinguish IBD from IBS because both can present in young adults and both
can be described as having increased frequency of mucus-covered stools. Features that
point towards IBS include lack of blood in stools, abdominal pain that is relieved with
defecation and alternating periods of constipation and diarrhea

6. A 67-year-old man comes to the emergency department 1 hour…

Right vertebral

Crossed signs of decreased pinprick sensation over opposite sides of face and body =
Pathology in the brainstem IPSILATERAL to side where face pinprick sensation is lost
Other aspects of patient’s presentation are consistent with Wallenberg syndrome
(stroke of lateral medulla), including vertigo, ataxia and weakness of the palate
Key idea: If patient with suspected brainstem stroke has symptoms that localize to
nucleus ambiguus (motor vagal nucleus that when damaged can present as dysphagia,
hoarseness, and dysarthria) then you are almost certainly dealing with a lateral
medullary syndrome
Although lateral medullary syndrome most often associated with stroke/hemorrhage
involving the Posterior Inferior Cerebellar artery (PICA), important to remember that
the PICA comes off the vertebral artery (reason why vertebral dissections can present
with lateral medullary syndrome)

7. A 25-year-old woman comes to the physician because…

Complete blood count

Appetite for non-nutritive substances (such as ice, hair, cornstarch on its own) = pica
Most commonly associated with iron de ciency anemia, with other clues in the stem
being the mild pallor on physical exam and leiomyomata uteri (which often lead to heavy
menstrual bleeding)

8. A 28-year-old woman at 28 weeks’ gestation…

Mitral stenosis

Diastolic murmur = Aortic regurgitation or mitral stenosis (assuming left-side of heart)


AR murmur = Early, diastolic, decrescendo murmur heard best at upper sternal border
MS murmur = Diastolic snap (“sharp sound after S2”) followed by a late diastolic murmur
heard best at the apex that can sometimes be described as increasing in intensity before
S1 (due to atrial kick pushing increased blood across valve right before valve closes)
Key idea: In setting of pregnant woman with new-onset dyspnea, always consider
mitral stenosis (increased cardiac output due to plasma expansion in pregnancy can
cause previously asymptomatic valvular defect to become symptomatic) and
pulmonary embolism (pregnancy is a hypercoagulable state)

9. A 37-year-old woman is brought to the emergency department…

Right optic nerve

Young woman + eye pain with ipsilateral vision problems = Optic neuritis
Optic neuritis (as the name implies) is due to in ammation of the associated optic nerve,
with further proof that optic nerve is involved being the relative afferent pupillary defect
(problem with light getting back to retina in this case)

10. A 70-year-old woman has had increasing abdominal pain…

Gram stain of abdominal uid


Patient with ascites who presents with diffuse abdominal pain/tenderness + fever +
leukocytosis = Spontaneous bacterial peritonitis
Diagnosis of SBP is actually typically made after paracentesis if the ascitic uid has >250
PMNs (neutrophils) per mL
Although we often think of Spontaneous Bacterial Peritonitis (SBP) as being a disease
exclusively seen in patients with cirrhosis, in reality it can affect patients with any
conditions that lead to ascites ( uid in the abdomen), with patients who undergo
peritoneal dialysis being at higher risk given the fact that signi cant opportunities for
contamination during catheter exchanges, etc.

11. A 32-year-old woman comes to the physician because of…

Thyroid-stimulating hormone

Dif culty of this question is trying to differentiate between normal stresses of being a
new mother vs. hypothyroidism
Features that point to hypothyroidism vs. normal stress include elevated cholesterol
levels, memory problems, and lethargy
Postpartum thyroiditis can present up to 1 year after giving birth and although it is a
fairly rare disease in real life, it is often tested on these exams
Cortisol = Cushing’s (gaining weight, hirsutism, etc.) or Adrenal insuf ciency (hypotensive,
hyperpigmented skin, craving salt, etc.)

12. A 67-year-old woman is hospitalized because of abdominal pain …

Intramural hematoma of the proximal small bowel

Patient recently started on anticoagulation with a severely elevated INR (therapeutic


range is 2-3) and rapidly dropping hemoglobin

13. A previously healthy 32-year-old woman comes to the physician …

Inhaled Beta-2 adrenergic agonist

3 common causes of a nighttime cough in an otherwise healthy person include asthma,


GERD and post-nasal drip
This patient’s presentation is most consistent with cough-variant asthma likely
exacerbated by recent URI because of the end-expiratory wheezes on physical exam and
decreased expiratory ow rate
GERD would more commonly lead to burning chest pain or sour taste, whereas post-nasal
drip would lead to cobblestoning of the throat and would be associated with other allergy
symptoms (runny nose, itchy eyes, etc.)

14. A 27-year-old primigravid woman at 38 weeks’ gestation ….

Recommend autopsy of the infant

Random question that could be approached through process of elimination (inconsiderate


to tell mother about becoming pregnant again right after losing child, don’t know risk of
recurrence without knowing cause, patient presented with baby already having lack of
fetal heart tones so nothing with hospital care led to outcome, etc.)

15. A 25-year-old woman comes to the physician for a routine…

Borderline personality disorder

Patient has persistent history of unstable relationships and self-injurious behaviors


Delusional disorder and adjustment disorder are transient pathologies
Major depressive disorder = Depressed mood and SIGECAPS

16. A 15-year-old girl is brought to the physician by her parents…

Negotiate a contract regarding medication compliance

Another random question that can be approached through process-of-elimination


Patient doesn’t need any changes to medication regimen because her problem is
compliance, cannot ethically refuse to see patient because of compliance, patient has no
psychiatric issues, and unethical for parents to punish patient for lack of compliance

17. A previously healthy 52-year-old man comes to the physician…

Insulin

Clinical picture consistent with Type 2 diabetes (older overweight man with polyuria,
nocturia and increased non-fasting serum glucose concentration)
Key idea: Early on in Type 2 diabetes, insulin levels are high because the pancreas is trying
to make up for the insulin-resistance
Key idea: Later on in Type 2 diabetes (on the scale of years after initial diagnosis), insulin
levels become low as the beta cells become burned out from overproducing insulin for so
long and secondary to amylin deposition within pancreas (amyloidosis)

18. A healthy 18-year-old woman comes for a routine health…


Testing for Neisseria gonorrhoeae and Chlamydia trachomatis

All sexually active women 24 and younger should receive STI testing, regardless of
whether they have been monogamous with only one partner
https://www.uspreventiveservicestaskforce.org/uspstf/document/RecommendationStat
ementFinal/chlamydia-and-gonorrhea-screening

19. A 5-year-old boy is brought to the emergency department 30 …

Hyaline casts

Hyaline casts are the most common type of urinary cast and are often seen in setting of
dehydration or vigorous exercise; they can be seen in healthy individuals
Erythrocyte casts = RBC casts = Glomerulonephritis (such as post-strep
glomerulonephritis)
Leukocyte casts = WBC casts = Pyelonephritis and acute interstitial nephritis

20. A 37-year-old woman comes to the physician because of progressive…

Pulmonary artery pressure

Classic presentation of mitral stenosis on NBME exams (rheumatic heart disease, worse
with pregnancy, murmur description with diastolic snap-murmur)
Over time, mitral stenosis will lead to increased pressures in the left atrium which will back
up into the lungs and the right side of the heart, eventually leading to right-sided heart
failure; this knowledge coupled with the right ventricular lift (which occurs because the RV
experiences hypertrophy secondary to increased pulmonary artery pressure) points to
elevated pulmonary artery pressure
Diastolic lling time decreased (takes longer for mitral valve to open) and LV end-diastolic
pressure also decreased/normal because of less blood ow from LA to LV

21. Two hours ago, a 24-year-old man had the sudden…

Tube thoracostomy

Tall, young men are prone to spontaneous pneumothorax, with the chest x-ray in this stem
showing a right-sided pneumothorax (vascular lines don’t extend to periphery and you can
actually see a clear outline of the collapsed lung within the lung eld)
A small (<2 cm), stable, asymptomatic pneumothorax can often be managed
conservatively with observation (https://www.nejm.org/doi/10.1056/NEJMoa1910775)
The patient in this stem is symptomatic and having dif culty breathing, so in this case we
would want to either perform a tube/needle thoracostomy
-ostomy = making a hole, whereas -otomy = cutting

22. A 42-year-old man is brought to the emergency department …

Vitamin B1 (thiamine)

Wernicke encephalopathy is due to thiamine de ciency, often seen in alcoholics (which is


a common NBME way of suggesting patient may have nutritional de ciency) and leads to
triad of confusion, ataxia and ophthalmoplegia
Key idea: Patients presenting to ED with altered mental status of unknown etiology often
given glucose, thiamine and naloxone, but a commonly tested concept on NBME exams is
that patients should get thiamine before glucose because glucose rst can induce
Wernicke encephalopathy in patient with underlying nutritional de ciency
Chlordiazepoxide would be used in patient with alcohol withdrawal or delirium tremens
(autonomic hyperactivity would be more prominent in presentation)

23. A previously healthy 10-year-old boy is brought to the physician…

Avoidance of the wooded area

Patient most likely has poison oak/ivy, which presents >1 day after exposure (Type 4
hypersensitivity) and will lead to a sharp line between rash and unaffected skin
Patients do not receive prophylaxis against poison oak/ivy
Cat scratch can lead to cat scratch fever, which leads to lymphadenopathy but will not
lead to vesicles/bullae

24. An otherwise healthy 4-month-old girl is brought to the physician…

No intervention is necessary

Presentation most consistent with a hemangioma of the skin, which commonly grows
during the rst year of life but involutes on its own within 5 years
https://clinicalgate.com/infantile-hemangiomas-and-vascular-malformations/

25. A 3-year-old boy is brought to the physician because of a 7-day…

Impaired phagocyte oxidative metabolism

Presentation most consistent with chronic granulomatous disease which is most


commonly X-linked (maternal uncle affected) and leads to recurrent skin abscesses with
aspiration showing neutrophils lled with bacteria (able to take up bacteria, but can’t
perform oxidative burst to kill bacteria)
X-linked immunode ciencies: Bruton agammaglobulinemia, Severe combined
immunode ciency, Hyper-IgM syndrome, Wiskott-Aldrich syndrome, Chronic
granulomatous disease

26. A 2-month-old boy is brought to the physician:

Language development: Normal // Psychosocial development: Normal

Language development: Babies “coo” around 2 months but don’t laugh until 3-4 months
Psychosocial development: Babies smile around 2 months, but won’t reach for toys until
3-4 months

27. A 72-year-old man comes to the emergency department after…

Corticosteroid therapy

Presentation most consistent with temporal arteritis (older patient with headache, jaw
claudication, elevated ESR and tender, enlarge temporal arteries)
Key idea: Because of risk for blindness and imperfect sensitivity of temporal artery biopsy
(because disease only affects section of temporal artery so biopsy may miss affected
segment), patients with suspected temporal arteritis should begin empiric treatment
with corticosteroids BEFORE temporal artery biopsy is performed
28. Four hours after undergoing a cesarean delivery at term…

Postoperative intra-abdominal hemorrhage

Patient’s blood pressure has dropped, pulse has gone up and hematocrit has dropped
relative to intraoperative values, pointing to bleeding after surgery

https://www.quora.com/How-much-blood-do-you-have-to-lose-to-start-feeling-dizzy-and-cause-a-
hypovolemic-shock

29. A 32-year-old woman comes to the physician because of a 1-year…

Histocompatibility human leukocyte antigen B27

Presentation most consistent with ankylosing spondylitis (young person with


in ammatory back pain (stiff for >1 hour upon waking and improves with movement) who
speci cally has involvement of sacroiliac joints)
HLA-B27 positivity is a risk factor for PAIR diagnoses (Psoriatic arthritis, Ankylosing
spondylitis, IBD related in ammatory arthritis and Reiter/Reactive arthritis)
Patient would have elevated ESR

30. A 32-year-old woman comes to the physician because she…

Polycystic ovarian syndrome

One of the most common causes of female infertility, especially in overweight women
Diagnosis requires 2 out of 3 of the following criteria: (1) Clinical and/or biochemical
hyperandrogenism (2) Oligomenorrhea (3) Polycystic ovaries on pelvic ultrasound /// Our
patient has clinical hyperandrogenism (acne vulgaris) and oligomenorrhea (irregular
menses)
Key idea: Although often associated with insulin resistance and ultimately associated
with development of Type 2 diabetes, insulin resistance is not formally included in
diagnostic criteria

31. A 57-year-old man is brought to the emergency department 30…

Intravenous calcium gluconate

Patient has missed several days of dialysis and was found down in his house (which can
lead to rhabdomyolysis), giving this patient 2 reasons to have metabolic acidosis with
hyperkalemia
Hyperkalemia can lead to abnormal heart rhythms that can lead to patient death, so
patients who have hyperkalemia with one of the following three features should rst
receive calcium gluconate to stabilize the cardiac membrane prior to trying to reduce
potassium levels: (1) Potassium > 7.0 (2) ECG changes (Peaked T waves, PR prolongation
and QRS widening, disappearance of P wave) (3) Rapidly rising potassium due to tissue
breakdown or tumor lysis syndrome

32. A 32-year-old man has had mild tremulousness, increasing…

Discontinuation of lorazepam

Alcohol and benzodiazepine withdrawal lead to similar clinical syndromes with increased
autonomic activity (tremors, anxiety, sweating, agitation, etc.) because prolonged use of
these medications leads to downregulation of GABA receptors, such that removal of
GABA agonists (alcohol, benzo’s) leads to relatively decreased inhibitory activity from
ligand binding to GABA receptors
Key idea: Withdrawal often leads to the opposite effects of intoxication with drug
(Example: Opiate use leads to constipation and pinpoint pupils, whereas opiate withdrawal
leads to diarrhea and mydriasis)
https://www.mdcalc.com/ciwa-ar-alcohol-withdrawal

33. A 20-year-old man comes for a routine health maintenance…

Myotonic muscular dystrophy

Autosomal dominant trinucleotide repeat disease (father also affected) that classically
leads to myotonia (dif culty relaxing muscles, most classically gripping) + CTG (Cataracts,
Toupee (early balding) and Gonadal atrophy
Key idea: These patients also commonly have psychosocial symptoms (irritability, tired
during day, etc.)
34. An 18-month-old boy is brought to the emergency department…

Passive hypersupination of the forearm

Presentation most consistent with radial head subluxation, which is where pulling on a
young child’s arm causes the radial head to be trapped within the annular ligament
Patient’s will classically be holding their arm in an extended and pronated position with
no visible deformity of elbow
Can be treated by either (1) Hyperpronation of forearm OR (2) Supination of forearm and
exion of elbow

35. A 4-year-old boy is brought to the emergency department…

Pulmonary contusion

Patient with blunt thoracic trauma who presents <24 hours later with symptoms
(tachypnea, tachycardia, hypoxia) and/or CT/CXR in ltrates over affected area (patchy,
alveolar)
Key idea: Important to differentiate from ARDS, which can have similar presentation but
often occurs 24-48 hours after trauma

36. An asymptomatic 57-year-old man comes to the physician…

Accumulation of lipids in the arterial wall

Abdominal bruit + hypertension = Renal artery stenosis


Most common causes of renal artery stenosis include atherosclerosis, bromuscular
dysplasia and polyarteritis nodosa
Our patient has clear risk factors (middle-aged man with signi cant smoking history and
likely other poor health behaviors) for atherosclerosis, which is the most common cause
overall
Fibromuscular dysplasia often seen in otherwise healthy young woman presenting with
new hypertension or signs of poor brain perfusion
Polyarteritis nodosa often seen with abdominal pain in a middle-aged man with hepatitis B
(or risk factors for Hep B)

37. A 20-month-old girl is brought to the physician because of …

Cystic brosis
Patient of Northern European descent with recurrent sinopulmonary infections, poor
growth (due to fat malabsorption) and signs of bronchiectasis (streaky lung densities with
hyperin ation, mild clubbing)
Important to distinguish from Kartagener’s syndrome, which would lead to dextrocardia
and would be less likely to lead to poor weight gain

38. A previously healthy 16-year-old boy is brought to the physician…

Pulmonary tuberculosis

Patient with clear exposure (Albania and other Eastern European countries are endemic
TB countries) showing signs of infection (cough and fever) with adenopathy
Pulmonary aspergillosis would more commonly be seen in somebody who is
immunocompromised (acute angioinvasive aspergillosis) or somebody with cystic brosis
or asthma (allergic bronchopulmonary aspergillosis)

39. A 24-year-old man is brought to the emergency department...

Observation in the emergency department

Presentation most consistent with marijuana intoxication (injected conjunctiva,


inappropriate laughter, increased appetite, paranoia) which is self-limited and managed
with observation

41. An 82-year-old woman is brought to the physician by her granddaughter…

Major depressive disorder

Differential for dementia in elderly patient includes: Alzheimer’s (chronic


neurodegenerative course), Vascular dementia (patient with ASCVD risk factors with
stepwise decline), Frontotemporal dementia (early personality changes often around 60
yo), Dementia with Lewy bodies (parkinsonism, visual hallucinations, uctuating
cognition), Normal pressure hydrocephalus (“wet, wacky and wobbly”) and
pseudodementias (depression, hypothyroidism, B12 de ciency)
In this patient with acute/subacute forgetfulness along with components of SIGECAPS
(trouble sleeping, decreased appetite, psychomotor retardation) and history of past
episodes of forgetfulness that were able to be treated with medication most consistent
with a major depressive disorder episode
Key idea: Alzheimer’s can also lead to signs/symptoms similar to Major depressive
disorder, but Alzheimer’s would have a chronic progressive course rather than an
episodic course as is seen in this question stem
42. A 42-year old computer science professor is brought to the…

Huntington’s disease

Patient between 40-60 years old often with family history of disorder or unknown family
history who presents with triad of chorea, depression and dementia
Key idea: Patients can also commonly have psychosis

43. A 77-year-old woman comes to the physician because of a 2-day…

Gallstone ileus

Patient has signs of a small bowel obstruction (abdominal distention, abdominal pain,
vomiting, high-pitched bowel sounds with air- uid levels on x-ray)
Most common causes of small bowel obstruction are adhesions, hernia and malignancy,
but this patient has air in the liver which is basically pathognomonic for gallstone ileus
(can also have air in the biliary tree)

References: Netter’s Atlas of Human Anatomy


44. A 67-year-old man is examined 3 days after undergoing…

Remove the urinary catheter

Most common types of hospital-acquired infections are central-line associated


bloodstream infections (CLABSI), Catheter-associated Urinary tract infections (CAUTI),
Surgical Site Infection and Ventilator-associated pneumonia
Best way to avoid Catheter-associated UTI is by removing the catheter as soon as
possible, which is appropriate in this patient because they have normal mental status and
are able to move
Key idea: Prophylactic antibiotics and bladder irrigation with antibacterial solutions can
increase the risk of developing resistant organisms without offering much bene t for
reducing infections

45. A 6-month-old girl is brought to the physician because of poor…

Patent ductus arteriosus

Leads to a widened pulse pressure (because blood ows from aorta to pulmonary
arteries during diastole, leading to reduced systemic diastolic pressure) with a continuous
to-and-fro murmur
Key idea: PDAs may also predispose to an increased risk of respiratory/lung infections

46. A program for the primary prevention of coronary artery…

Incidence

Incidence > Prevalence because you are concerned about development of new cases
(primary prevention)
Incidence = Number of new cases over a time period
Prevalence = Number of cases in the population
Mortality, case fatality and hospitalization would be more re ective of tertiary disease
prevention efforts

Exam section 2:

1. A 22-year-old woman comes to the physician for a health…

Administration of hepatitis B vaccine


Patients who are adequately vaccinated against hepatitis B vaccine will have negative
HBsAg, positive HGsAb and negative Hepatitis B core antibody (positive Hep B core
antibody = past or current infection)
Patient is adequately protected against Hepatitis A due to presence of IgG antibodies,
which can be the result of vaccination or past infection
Key idea: Indications for Hep A vaccination are chronic liver disease or increased risk of
contracting disease (men who have sex with men, IVDU, travel to country where hepatitis
A is prevalent, etc.)

2. A 5-month-old boy is brought for a follow-up examination. He…

Compression of the airway by a vascular ring

Patient has persistent wheezing despite treatment with bronchodilators, meaning that
asthma is unlikely
3 causes of chronic stridor/wheezing in a newborn/infant include laryngomalacia
(inspiratory stridor/wheezing that worsens when feeding, crying or supine and improves
when prone), vascular ring (biphasic stridor/wheezing that improves with neck
extension) and airway hemangioma (worsening biphasic stridor over rst year of life in
patient with concurrent skin hemangioma)
Patients with proetin-induced allergic proctocolitis secondary to milk consumption often
have bloody stools in setting of other allergic conditions (allergies, eczema or asthma)

3. A 37-year-old man is brought to the emergency department 6…

Laparotomy

Key idea: Peritonitis (rigidity, guarding) + Free air under diaphragm = Emergency
laparotomy!!!!!

4. An asymptomatic 32-year-old man comes for a routine health…

Selective IgA de ciency

Selective IgA de ciency commonly leads to sinopulmonary and GI infections (especially


Giardia) and pathognomonically anaphylaxis to blood products (because patient
develops anti-IgA antibodies and reacts against IgA in blood products)
Key idea: Most common immunode ciency and is most commonly asymptomatic

5. Five weeks after vaginal delivery of a healthy full-tern newborn…


Admission to the hospital for treatment

Peripartum mood disorders are commonly seen within 4 weeks delivery


Most common is Postpartum blues (50-75% of pateints) who will have depressed affect
and fatigue, but will have resolution within 2 weeks delivery
The most severe (and least common) peripartum mood disorder is postpartum
psychosis, which often leads to delusions, hallucinations and thoughts of suicide or
infanticide; these patients require immediate hospitalization due to high risk of injury to
self or baby

6. A 17-year-old boy comes to the physician because of a 2-day history…

Inadequate production of insulin

Classic presentation of diabetic ketoacidosis in the setting of Type 1 diabetes (polyuria,


polydipsia, tachypnea (to compensate for metabolic acidosis), fatigue/weakness, elevated
glucose (often 300-500 mg/dL) and elevated ketones)
Younger patients can also develop type 2 diabetes, but that would more commonly be
seen in a patient with elevated BMI and Type 2 diabetes almost never presents as DKA

7. A 62-year-old woman is admitted to the hospital because of…

Glucocorticoid-induced demargination and storage pool release of segmented


neutrophils

Patient presented with a COPD exacerbation without overt infection (normal WBC with
normal neutrophil count, afebrile) and then after receiving IV methylprednisolone (strong
systemic steroid) developed a neutrophilic leukocytosis despite clinical improvements
Key idea: Two common causes of a neutrophilic leukocytosis are infection and systemic
steroid use

8. A previously healthy 4-year-old boy is brought to the physician…

Unable to access media for this question, but patient likely had a no murmurs or a soft
systolic ejection murmur in the setting of a clinical URI
CHF = S4 heart sound and crackles/rales (patient would have signs of edema)
Idiopathic pulmonary hypertension: Loud S2 and RV heave (patient would have chronic
progressive dyspnea)
Pericarditis: Three-component friction rub (history would involve positional chest pain)
Ventricular septal defect: Common heart abnormality that would lead to a systolic
murmur over the left lower sternal border, but would not explain the patients fever, cough
and runny nose

9. A 3-month-old boy is brought for a well-child examination…

Glycogen storage disease, type 2 (Pompe’s disease)

Classically leads to cardiomegaly and hypotonia (including poor head control) in a young
child
Muscular dystrophy would often present later on (toddler) and wouldn’t lead to heart
symptoms early on in disease course
GM1 gangliosidosis [similar to Tay-Sachs (progressive neurodegeneration)] and infant
botulism wouldn’t lead to heart symptoms

10. A previously healthy 4-year-old girl is brought to the physician…

Arthrocentesis

In children with new development of joint pain, two major diagnoses to consider are septic
arthritis and transient synovitis
Our patient’s presentation more consistent with septic arthritis due to fever and refusal
to bear weight, and therefore patient requires immediate drainage of joint space even
before administering antibiotics because of risk for persistent damage to joint
Transient synovitis: Afebrile, history of previous viral URI, normal ESR, normal WBC,
normal joint aspiration, self-limited condition

12. A 24-year-old woman comes to the physician because of constant, severe…

Fibromyalgia

Fibromyalgia classically leads to chronic widespread pain + tenderness at trigger points


(pathognomonic) + normal labs and is often seen in anxious young patients (which is
suggested by this patient’s diagnosis of irritable bowel syndrome)
Polymyositis: Proximal muscle weakness + elevated muscle enzymes + other signs of
in ammation (ESR, CRP, etc.)
Polymyalgia rheumatica: Older patients with stiffness > pain and elevated markers of
in ammation (ESR, CRP); important association with giant cell arteritis

13. A 67-year-old man is brought to the emergency department 4…

Ruptured aortic aneurysm


Older patient with sudden onset back pain unrelated to activity with signs of blood loss
(pale, tachycardic, diaphoretic) who also has calci cations anterior to vertebral bodies on
X-ray (aorta sits just anterior to vertebral bodies)

14. A 25-year-old woman comes to the physician for evaluation 1 …

Latex products

Patient with new job as a medical assistant (where she likely needs to where latex gloves)
has developed a new rash on her hands, most consistent with allergic contact dermatitis
Key idea: Latex allergy seen in 1-5% of population and is seen at higher rates closer to
10% in individuals with chronic occupational exposure to latex (medical workers, etc.)

15. A previously healthy 16-year old boy is brought to the…

Viral infection

Patient presents with signs of encephalitis (altered mental status, emotional lability, fever,
etc.) with CSF ndings consistent with a viral infection (elevated WBC count with
lymphocytic predominance, elevated protein and normal glucose), most consistent with
HSV encephalitis
Fungal infection CSF: Elevated WBC count with lymphocytic predominance with
decreased glucose
Bacterial infection CSF: Highly elevated WBC count (>1000) with neutrophilic
predominance with decreased glucose
Normal CSF ndings: WBC 0-5, Glucose 40-70 (2/3 serum glucose levels), Protein < 40
Key idea: Patients with suspected HSV encephalitis should be started on empiric IV
acyclovir while awaiting PCR results and/or before brain MRI

16. A 52-year-old woman comes to the physician because of…

Adjustment disorder with anxiety

Patient with psychosocial distress causing impairment following a discrete stressor who
does not meet criteria for more serious psychiatric diagnoses
Key idea: Adjustment disorder develops within 3 months of identi able stressor and
should last no longer than 6 months
Key idea: Treated with psychotherapy (CBT) +/- adjunctive pharmacotherapy for anxiety,
insomnia, etc.

17. An 8-year-old girl with type 1 diabetes mellitus is brought to…


Infusion of 0.9% saline

Young patient involved in a motor vehicle accident who initially developed thigh pain and
now has impaired consciousness with signs of hemodynamic instability most consistent
with traumatic bleeding into the thighs, leading to hypovolemia
Key idea: Hematocrit can be normal during acute hemorrhage due to loss of equal parts
of red cells and plasma, and may not drop until patient has received volume resuscitation
Key idea: Potential sites of bleeding in setting of traumatic hemorrhagic shock that can
accommodate lots of bleeding are “Blood on the oor and 4 more”: (1) Floor (obvious
external bleeding) (2) Chest (3) Abdomen (4) Pelvis (5) Thigh

18. A 52-year-old Cantonese woman comes to the physician…

Share the results with the husband only

Patients have the autonomy to decide whether or not they want to hear results of their
own medical tests

19. A 21-year-old college student comes to the physician because…

Topical retinoic acid

Acne treatment in order of increasing severity/potency: Topical retinoids +/- benzoyl


peroxide —> Topical antibiotics —> Oral antibiotics —> Oral isotretinoin
No good evidence that restricting milk products or sugar reduces acne

20. A 67-year-old woman has had anorexia, fatigue, and weakness…

Multiple myeloma

Multiple myeloma = Bone pain (often in the back) + Lytic bone changes on imaging +
Anemia (due to plasma cells crowding out other hematopoeitic cells in bone marrow) +
Elevated calcium concentration (due to lytic bone lesions) + Rouleaux formation on
peripheral smear (due to sticky immunoglobulin proteins causing RBCs to stack up)
Key idea: If you see a patient with signs of kidney disease (elevated creatinine/BUN) and
hypercalcemia, you should have a high index of suspicion for multiple myeloma because
most other forms of kidney disease lead to hypocalcemia (due to binding of phosphate to
calcium and reduced vitamin D activation)

21. A 32-year-old woman is brought to the emergency department…


Hemothorax

Opacities involving an entire lung lobe most consistent with collapsed lung tissue
(atelectasis) vs. accumulation of blood/ uid/pus within that space
In this patient involved in a traumatic situation + signs of hemodynamic instability
(tachycardic, borderline hypotensive) with complete opaci cation of right hemithorax and
absent breath sounds on right most consistent with hemothorax
Pneumothorax would lead to lung lucency (increased air), diaphragmatic hernia almost
always left sided (due to liver protecting right diaphragm) and ruptured bronchus often
leads to persistent pneumothorax and/or pneumomediastinum

23. A 42-year-old man comes to the physician because of progressive swelling…

Constrictive pericarditis

Constrictive pericarditis = Patient with history of chest radiation + right-sided heart


failure + Pericardial knock (early diastolic sound that can mimic S3 resulting from loss of
pericardial elasticity during ventricular lling) + Kussmaul’s sign
Common causes of constrictive pericarditis include radiation therapy, cardiac surgery,
connective tissue disorder, tuberculosis and viral pericarditis
Jugular venous distention that increases with inspiration = Kussmaul’s sign = Constrictive
pericarditis, right heart failure/infarction, restrictive cardiomyopathy
Key idea: Patients with constrictive pericarditis or other forms of global diastolic
dysfunction (restrictive cardiomyopathy) more commonly present with signs of right
heart failure because the right heart is responsible for lling whereas the left heart is
responsible for squeezing
Cor pulmonale = Right-sided heart failure due to primary lung disease (COPD, ILD,
etc.)

24. A 35-year-old man is brought to the emergency department because…

Na 140 / Cl 80 / K 2.5 / HCO3 40

Vomiting classically leads to a hypochloremic, hypokalemic metabolic alkalosis


Key idea: Hypochloremia and metabolic alkalosis due to loss of HCl (stomach acid)
whereas hypokalemia (and a small component of metabolic alkalosis) due to contraction
alkalosis phenomenon in the collecting duct (reabsorbing Na and H20 in exchange for H+
and K+)

25. A previously healthy 52-year-old man comes to the emergency department…


Fluid restriction

Patient with signi cant smoking history presents with right hilar mass + hyponatremia =
SIADH due to small cell lung cancer
Note: Hiccups could either be secondary to hyponatremia or could be due to lung mass
irritating the diaphragm
Mild symptoms of SIADH (sodium >120, lethargy): Fluid restriction +/- salt tablets
Severe symptoms of SIADH (sodium <120, seizures, coma): Hypertonic (3%) saline
Key idea: Don’t want to correct hyponatremia too quickly in order to avoid osmotic
demyelination syndrome (“low to high, the pons will die”)

26. A 27-year-old woman comes to the physician because of…

Irritable bowel syndrome

Young, otherwise healthy patient with intermittent constipation and intermittent diarrhea
with no in ammatory/systemic signs (fever, weight loss, etc.) and normal exam
Key idea: Patients with IBS can have foul-smelling, mucus-coated stools
Key idea: Features inconsistent with IBS include rectal bleeding, nocturnal abdominal
pain, weight loss, grossly abnormal lab ndings (anemia, etc.), and older age

27. A 62-year-old woman comes to the physician because of generalized…

Avoidance of analgesics

Analgesic nephropathy (chronic interstitial nephropathy): Patient with chronic


osteoarthritis treated with NSAIDs (and tylenol) who presents with signs of renal
insuf ciency (elevated creatinine and BUN) with mild proteinuria and sterile pyuria

28. A 37-year-old woman comes to the physician because of an itchy…

Medication adverse effect

Presentation most consistent with acute interstitial nephritis, which leads to


combination of rash, AKI, pyuria, hematuria, WBC casts, eosinophilia and urinary
eosinophils
Almost always caused by medication side effect, with common culprits being the 5 P’s
(PPIs, Pain-free (NSAIDS), Pee (Diuretics), Penicillins and other antibiotics (especially
sulfonamides such as in this patient) and rifamPin

29. An 87-year-old nursing home resident with dementia, Alzheimer’s…


Decreased gag re ex

Patient’s with Alzheimer’s disease are very prone to aspiration pneumonia both due to
dysphagia and problems protecting their airway due to decreased gag re ex and
impaired consciousness
Clues to aspiration pneumonia = Older patient (often demented) who presents with
pneumonia in the right lung base (most common location due to right mainstem
bronchus being wider, shorter and more vertical)
https://www.ncbi.nlm.nih.gov/pubmed/14577062

30. A 13-year-old girl is brought for a well-child examination…

Discussion of pregnancy prevention

Patient has normal sexual development and is not sexually active, so no reason for any of
the lab tests or procedures
Important to discuss pregnancy prevention in adolescents, even in patients who are not
yet sexually active

31. A 57-year-old woman with breast cancer comes to the physician…

Spinal cord decompression and cervical stabilization

Patient has signs/symptoms concerning for cervical spinal cord compression (neck pain
with tenderness upon palpation, diffuse hyperre exia) secondary to spinal metastases
Key idea: Acute spinal cord compression is a neurological emergency and requires
immediate surgical decompression

32. A 5-week-old boy is brought to the physician because…

Hypertrophic pyloric stenosis

Classically leads to non-bilious (stenosis occurs above the duodenum) projectile


vomiting in a newborn at 2-6 weeks of age that is refractory to dietary changes
Key idea: In questions asking about management, you want to rst correct uid and
electrolyte abnormalities before proceeding with a corrective pyloromyomectomy

33. A previously healthy 44-year-old woman, gravida 4, para 4…

Stress incontinence
Classically leads to urine leakage with increased intra-abdominal pressure due to
sneezing, coughing, laughing, and exercising
Risk factors for stress incontinence include obesity and multiple vaginal deliveries (as is
the case with this patient)
Key idea: All types of urethroceles are associated with stress incontinence because
urethroceles can disrupt the normal positioning/function of urinary sphincters

34. One day after an uncomplicated spontaneous vaginal delivery…

Over ow incontinence

Classic presentation of postpartum urinary retention (inability to void or small volume


voids for at least 6 hours after vaginal delivery with incomplete emptying of bladder)
complicated by over ow incontinence (dribbling of small amounts of urine)
Key idea: Post-void residual > 150 mL is a sign of urinary retention
Key idea: This speci c presentation managed by intermittent catheterization

35. A 32-year-old woman, gravida 2, para 2, comes to the physician …

Anovulation

Differential for secondary amenorrhea = Pregnancy, Menopause, PCOS, hypothyroidism,


prolactinoma, primary ovarian insuf ciency, Asherman syndrome, functional
hypothalamic amenorrhea, etc.
First step in setting of secondary amenorrhea (amenorrhea for >3 months in woman with
previously regular menses or >6 months in woman with previously irregular menses) is to
obtain a pregnancy test
Presence of withdrawal bleed after progesterone administration con rms that the uterus
is undergoing a normal proliferative phase and therefore eliminates estrogen
de ciency (primary ovarian failure, menopause, hyperprolactinemia, hypothyroidism,
functional hypothalamic amenorrhea), endometrial abnormalities (Asherman’s) and
Outlet tract abnormalities (imperforate hymen) as the cause of secondary amenorrhea
Therefore, secondary amenorrhea with progesterone withdrawal bleed indicates that the
cause is anovulation, which is most commonly caused by PCOS

36. A healthy 21-year-old college student comes for an annual…

Pap smear

USPTF recommends cervical cancer screening every 3 years in women aged 21 to 29


years old regardless of sexual activity; For women 30-65 years old, can screen with
cervical cytology alone every 3 years, high-risk HPV testing alone every 5 years or
combined cytology and high-risk HPV every 5 years
https://www.uspreventiveservicestaskforce.org/uspstf/recommendation/cervical-cancer-
screening

37. An asymptomatic 47-year-old man comes for a preemployment…

Serum lipid studies while fasting

Key idea: Abnormal total cholesterol levels should always be followed by fasting LDL
levels because treatment is based on LDL and HDL levels/thresholds

38. Four weeks after a low transverse cesarean delivery…

Reassurance

Key idea: Pulling effect/feeling is common complaint in the post-partum for patients who
underwent C-section and is due to healing brosis around the wound site
No fever, wound erythema/purulence, or feeling of hematoma/seroma beneath the wound
site
Note: Women are often advised to avoid exercise or sexual activity for at a minimum of 6
weeks after C-section

39. A 23-year old woman has pain, cramping, and swelling of the…

Hypercoagulable state of pregnancy

Presentation is most consistent with a Deep Vein Thrombosis, which is a feared


complication of pregnancy
Key idea: Pregnancy is a hypercoagulable state, which likely evolved because we don’t
want women bleeding out during delivery

40. A 62-year-old man has had the gradual onset…

Tricuspid regurgitation

Murmur that is loudest on inspiration = Right-sided valvular defect (tricuspid or


pulmonic) —> Location of murmur = Tricuspid valve —> Holosystolic tricuspid valve
murmur = Tricuspid regurgitation
Large wave occurring with S2 caused by blood aberrantly moving from RV to RA and up
into SVC during systole
Eventually leads to RV hypertrophy due to increased preload (lifting systolic murmur of
the sternum) and eventually right heart failure (congestive hepatomegaly, lower
extremity edema, etc.)

42. A 25-year-old woman comes to the emergency department…

Idiopathic intracranial hypertension

Idiopathic intracranial hypertension = Pseudotumor cerebri with risk factors being Female
TOAD (Female, Tetracyclines, Obesity, vitamin A and Danazol)
Often presents in young overweight women with combination of headaches, vision loss
(enlarged blind spot), and pulsatile tinnitus with signs of increased intracranial pressure
(CN 6 palsy, papilledema, lumbar puncture ndings of increased opening pressure
[>250])
Key idea: Treat with weight loss and acetazolamide

43. A 25-year-old man with a history of intravenous drug use…

Cryptococcal meningitis

Patient with risk factors for immunosuppression (IVDU can lead to HIV due to needle
sharing) presents with signs of increased intracranial pressure (papilledema and
weakness of lateral rectus muscle [CN 6 palsy]) and meningitis (headache, fever, neck
stiffness) with CSF ndings consistent with a fungal etiology (elevated WBC count with
lymphocytic predominance, low glucose)
Key idea: Cryptococcal meningitis classically leads to meningitis symptoms + signs of
elevated ICP + umbilicated skin lesions
Note: Elevated ICP preferentially leads to a CN 6 palsy is a contested issue with two
potential explanations: (1) It has the longest intracranial course and therefore is prone to
stretching in setting of elevated ICP (2) CN 6 emerges straight from brain stem rather
than obliquely/transversely like other cranial nerves, and is therefore more prone to
compression by backward brain displacement in setting of elevated ICP

44. A 7-year-old girl is brought to the physician in September…

Throat culture

Pharyngitis and fever without viral URI symptoms in a young child always concerning for
Strep throat, which requires rapid strep test followed by throat culture if negative [NO
EMPIRIC TREATMENT IN KIDS]
Key idea: In adults, patients risk of Strep throat is strati ed based on Centor criteria [(1)
Fever by history (2) Tender anterior cervical lymphadenopathy (3) Tonsillar exudates (4)
Absence of cough], and with patients receiving no testing (scores 0-1), rapid strep test
followed by throat culture if negative (2-3) or empiric antibiotics (4+)

Key idea: In contrast, CENTOR criteria are not applied to children, with children who have
potential signs of strep throat (exudative pharyngitis, palatal petechiae) and NO viral
symptoms (cough, rhinorrhea, conjunctivitis, oral ulcers) receiving a rapid antigen test
followed by throat culture if rapid strep test is negative (i.e. all children receive
diagnostic tests and none are treated empirically)

45. After an uncomplicated laparoscopic cholecystectomy, a…

Reinsertion of a Foley catheter

Presentation most consistent with acute urinary retention given demographic features
(older man who is post-op) who presents with suprapubic tenderness (abdominal pain)
and anuria unresponsive to uids
Managed via catheter insertion and urinalysis (rule out UTI)
Patient is unresponsive to 1 L, so unlikely that another 1 L will help
Patient is producing no urine, so diuretic is unlikely to help

46. A 20-year-old man is brought to the emergency department…

Inadequate dissipation of body heat

Presentation most consistent with heat stroke, which leads to elevated temperature and
CNS dysfunction +/- other organ damage in a patient participating in strenuous activity
in hot/humid weather and is caused by impaired thermoregulation
Key idea: Managed with aggressive cooling and uid/electrolyte correction with NO role
for antipyretic therapy
Contrast with heat exhaustion, which leads to elevated temperature with no CNS
dysfunction and is caused by inadequate salt and water replacement

Exam section 3:

2. An 18-year-old man comes for an examination prior to participation…

Germinal cell tumor

Presentation most consistent with testicular cancer, with germ cell tumor being the
most common cause
Testicular mass that does not transiluminate = Cancer or varicocele
Varicocele would not lead to a discrete mass, but rather a “bag of worms” feeling and it
would decrease in size when patient placed in supine position due to increased venous
return to heart
Testicular mass that does transiluminate = hydrocele or spermatocele
Key idea: Oftentimes patients discover masses after trauma to that area because they
are paying more attention to the area, so don’t automatically write-off the mass as being
the result of trauma (for example, many NBME questions have patients nding breast
masses after trauma to area)

3. A 50-year-old woman has had progressive dyspnea over the past…

Echocardiography

Presentation is most consistent with cardiac tamponade (chest pain + hypotension +


pulsus paradoxus + jugular venous distention + “water-bottle” heart on CXR) most likely
due to breast cancer recurrence/metastasis to pericardium (2nd most common cause
of malignant pericardial effusion)
Next best step would actually probably be pericardial window, but of the choices
echocardiography is most appropriate
Pericardiocentesis: Drainage of pericardium
Thoracentesis: Drainage of pleural space
Paracentesis: Drainage of peritoneal cavity

4. A county health of cer investigates an outbreak of illness…

Staphylococcus aureus

Causes of rapid-onset food poison (< 6 hours) are due to ingestion of pre-formed toxin
and are most commonly caused by Staph aureus (cream-based food such as egg salad) or
Bacillus cereus (reheated rice syndrome)
Clostridium perfringens (improperly cooked/stored meat) is another cause of food
poisoning, but often occurs 6-24 hours after ingestion because patients ingest spores
which need germinate in digestive tract to secrete toxin

5. A 38-year-old woman, gravida 2, para 1, at 38 weeks’…

Nonstress test

If a patient presents with decreased fetal movement, rst perform a non-stress test to
look at fetal heart rate (110-160 beats/min), variability (consistent and not too much,
but not none at all) and acceleration (15/15 2 in 20: heart rate should rise by at least 15
beats/min for at least 15 seconds at least 2 times in 20 minutes)
If that test result is abnormal, then perform a non-stress test with acoustic
stimulation and assess the same way you assess non-stress test (rate, variability and
acceleration)
Abnormal non-stress test with stimulation means you should perform a biophysical
pro le, which is similar to APGAR and assigns a score from 0 to 2 in 5 categories (non-
stress test result, amniotic uid index, fetal breathing, fetal movements and fetal tone)
Unequivocal biophysical pro le (greater than 2 but below 8) should be followed up
with contraction stress test to look for late decelerations (decelerations begin at same
time that contractions occur) and/or bradycardia (HR < 110 bpm)

6. A 52-year-old woman comes to the physician because of a 3-month…

Intraductal papilloma
Most common cause of unilateral bloody nipple discharge, and often presents without
mass or lymphadenopathy

7. A 17-year-old boy is brought to the emergency department by…

PCP

PCP intoxication characterized by agitation, nystagmus and aggressive behavior

8. A previously healthy 62-year-old man comes to the emergency…

CT scan of the abdomen

Presentation most consistent with diverticulitis, which classically leads to fever + LLQ
pain in an older person /// First step in work-up is abdominal CT

9. A 47-year-old man comes to the physician because of fever and chills…

Carbamazepine therapy

WBC differential shows severe neutropenia even though patient likely has an active
infection, which is most consistent with a diagnosis of agranulocytosis
3 drug classes that are associated with agranulocytosis include (1) Carbamazepine
(antiseizure) (2) Clozapine (antipsychotic) and (3) Methimazole/Propylthiouracil (anti-
thyroid)
Key idea: Classic presentation of agranulocytosis is sore throat + fever + low neutrophil
count
Note: Patient also has a mildly elevated MCV, which is likely secondary to signi cant
alcohol exposure

10. A 62-year-old woman comes to the physician because of…

Malabsorption

Presentation most consistent with lactose intolerance (bloating, abdominal pain and
intermittent diarrhea that have worsened with increased dairy intake) which is a form of
malabsorption because lactose cannot be broken gown into glucose and galactose
monomers
Impaired intestinal motility would lead to decreased bowel sounds
In ammatory process would likely lead to fever +/- blood in stool
11. A 42-year-old man comes to the emergency department because…

Amphotericin B

Presentation is most consistent with Cryptococcal meningitis (headache, neck stiffness,


fever, signs of elevated intracranial pressure, CSF consistent with fungal infection (low
glucose with lymphocytic predominant WBC elevation) and they tell us that the
cryptococcal antigen assay is positive
Patients are often treated with Amphotericin + Flucytosine initially for about 2 weeks
and then switched over to uconazole for another few months

12. Two days after admission to the hospital for treatment of severe…

Acute tubular necrosis

Patient presented with hemodynamic instability secondary to sepsis and developed an


AKI (creatinine concentration increased by at least 0.3 mg/dL)
Two most likely causes of AKI given hypotension are pre-renal vs. Acute tubular necrosis
Features that point to ATN in this case include BUN:Cr ratio < 15, FeNa > 2% and muddy
brown casts on urinalysis
In contrast, pre-renal AKI would lead to BUN:Cr ratio > 20 and a FeNa < 1% (both due to
increased reabsorption of Na+ from renal tubules in order to compensate for
hypovolemia)

13. A 77-year-old woman is brought to the physician…

Memory loss

NBME loves to ask about complaints in elderly patients to assess whether you know
which changes are physiological vs. pathological (same with newborns and developmental
milestones)
In this patient, all of the ndings are normal except for the memory loss because it has
only been present for 1 month, which does not correspond with the typical picture of
Alzheimer’s
Patients with suspected cognitive impairment are often worked up with cognitive tests
(MOCA, MMSE, Mini-Cog), Lab testing (CBC, B12, TSH, BMP +/- syphilis) and Imaging (CT
or MRI of brain)
Key idea: Mild cognitive impairment = short-term memory/cognitive problems that are
concerning to patient/family but that DON’T affect daily life or ADLs (using phone, driving,
cooking, etc.)
14. A 2-year-old boy is brought to the physician because of fever…

Intravenous immunoglobulin infusion

Patient has an X-linked condition (2 maternal uncles affected) that leads to recurrent
infection with encapsulated bacteria (Strep pneumoniae) with labs showing normal
lymphocyte count with very low immunoglobulins, most consistent with X-linked
(Bruton’s) agammaglobulinemia
Patient should receive treatment of acute infection and IV immunoglobulin infusion in
order to make up for lack of endogenous immunoglobulin production

15. A 67-year-old man with long-standing signs and symptoms of…

Protein 2.5 / Glucose 90 / Leukocyte count 2000 / Segmented neutrophils 60% /


Monocytes 40%

Pleural effusion is most likely transudative, so we would expect low protein, relatively low
leukocyte count and a glucose near normal serum levels, which is most consistent with
selected answer
Key idea: For new pleural effusions, often can determine transudative vs. exudative using
Light’s criteria where if they have at least 1 of 3 positive then it is exudative: (1) Pleural
protein:Serum protein > 0.5 (2) Pleural LDH:Serum LDH > 0.6 (3) Pleural LDH > 2/3 normal
serum LDH levels

16. A 57-year old man is brought to the emergency department…

Carotid ultrasonography

Patient with vasculopathic risk factors (hypertension and poorly controlled diabetes) who
presents with recurrent embolic phenomenon to brain all from the same side (will lead to
ipsilateral visual problems (amaurosis fugax) and contralateral weakness (due to MCA
involvement))
Key idea: Amaurosis fugax is highly associated with carotid artery disease

17. A previously healthy 27-year-old woman is brought to the …

Electromyography and nerve conduction studies

Presentation most consistent with Guillain-Barre syndrome (development of ascending


weakness, decreased re exes, and sensory symptoms over the course of days-weeks in a
young patient with a recent URI or GI illness)
Best tests to con rm Guillain-Barre are lumbar puncture (showing increased protein with
normal leukocytes (albuminocytologic dissociation) or electromyography/nerve
conduction studies because the underlying pathophysiology is a demyelinating disease

18. A previously healthy 42-year-old carpenter comes to the…

Median nerve at the wrist

Distribution most consistent with median nerve, and the history of patient with recurrent
use of hands (carpenter) with pain and numbness at night most consistent with carpal
tunnel syndrome, which is due to compression of median nerve at the wrist
Key idea: Carpal tunnel syndrome leads to spared sensation to the palm because the
palmar cutaneous branch enters the hand external to the carpal tunnel

https://www.anatomynote.com/human-anatomy/nerves-system/median-nerve-ulnar-nerve-radial-nerve-
innervation-area-in-hand/

19. A previously healthy 37-year-old man comes to the physician…

Ulnar nerve at the elbow

Patient has numbness and weakness in the distribution of the 5th nger and 1/2 of the
4th nger, which is consistent with the ulnar nerve
Key idea: Most common site of ulnar nerve compression is at the elbow because it can be
compressed when the elbows are resting while sitting at a desk, driving, etc., and is
especially relevant in this patient who has paresthesias with compression of the cubital
tunnel (which is where ulnar nerve passes by the elbow)

https://teachmeanatomy.info/upper-limb/nerves/ulnar-nerve/

20. An 18-year-old man comes to the physician for an initial…

Ask about suicidal feelings

Patient has presentation consistent with somatic symptom disorder, although this patient
also has features of psychosocial distress (hopeless, fatigue, etc.) concerning for
depression with somatic symptoms
Key idea: In patients with depression, always ask about suicidal ideation/plan/intent

21. A previously healthy 67-year-old man is admitted to the hospital…

Small cell carcinoma

Patient who presents with non-speci c lethargy, confusion and muscle cramps found to
have severe hyponatremia with a lung mass most consistent with Small cell carcinoma
complicated by SIADH

22. The genetic disease institute at a university hospital…

If we decrease the prevalence of a disease in a population, then the ratio of true


positives:false positives tips towards false positives (we would expect fewer true
positives), leading to reduced PPV
At the same time, a decrease in disease prevalence causes the ratio of true
negatives:false negatives to tip towards true negatives (we would expect more true
negatives), leading to increased NPV
Key idea: Sensitivity and speci city are not affected by disease prevalence
(characteristics of test)

23. An asymptomatic 32-year old woman comes for a routine…

Weight-loss program

Most important risk factor for back pain and osteoarthritis is obesity
Answer would not be limiting physical activity because patients with acute back pain are
actually advised to remain active and to avoid bed rest

24. A 32-year-old man receiving intensive chemotherapy for…

Pneumocystic jiroveci (formerly P. carinii)

Immunocompromised patient (intensive chemotherapy) develops fever with diffuse


reticular opacities on chest x-ray most consistent with PCP
Key idea: Patients often have elevated LDH levels on lab testing and treatment consists
of TMP-SMX +/- prednisone (if patient is hypoxic)

25. A 64-year-old woman has moderately severe postoperative pain…

Patient-controlled intravenous morphine

Patient is in severe pain after a major surgery, and therefore would bene t from use of
opiate medications
A common form of pain control in the hospital is placing patients on patient-controlled IV
morphine where the patient is allowed to self-administer doses, but the physician team
can put limits as to the frequency and total amounts of morphine that are administered
Aspirin-codeine is more appropriate for mild-moderate pain, and transcutaneous fentanyl
patches are a form of long-acting opiates that are not used in the post-op patient but
rather in patients with chronic pain needs (example: patient with metastatic cancer with
painful bony mets)

26. Three days after being hospitalized for treatment of a hip…

Heparin therapy

Post-op patient who has been bed-bound for multiple days has developed dyspnea and
hemoptysis with V/Q scan showing areas of mismatch, most consistent with acute
pulmonary embolism
Key idea: The rst step in management of acute PE is administration of an anticoagulant,
most often heparin (assuming patient has no absolute contraindications such as active
bleeding, hemorrhagic stroke, etc.)
Some patients who are hemodynamically unstable (which is not the case here) will also
receive thrombolysis either through use of brinolytics or interventional radiology
procedures, but even these patients are often rst started on a heparin drip while
de nitive diagnostic tests (such as CT-PE) are performed

https://www.uptodate.com/contents/image?
imageKey=PULM%2F57249~PULM%2F99762&topicKey=PULM%2F8265&source=see_link

27. A 32-year-old woman comes to the physician because of …

Clue cells
Patient has grayish vaginal discharge with pH > 4.5, which is most consistent with
bacterial vaginosis, and therefore we would expect clue cells
3 main forms of vaginal infection and de nitive features include (1) Bacterial vaginosis
[gray shy discharge, pH > 4.5, no cervical/vaginal erythema, clue cells, treat with
metronidazole or clindamycin] (2) Candidal vaginitis [white cottage cheese discharge, pH
< 4.5, cervical/vaginal erythema, pseudohyphae and budding yeast, treat with oral or
topical -azole drugs] (3) Trichomonal vaginitis [greenish-yellowish discharge, pH > 4.5,
cervical/vaginal erythema, motile trichomonads, treat with metronidazole

28. A previously healthy 67-year-old man comes to the physician…

Daily exercise program

Key idea: Patients with claudication (such as this patient who has signi cant
vasculopathic risk factors and has leg pain with exercise) should rst be optimized with
smoking cessation + daily exercise program + statin + aspirin
Key idea: Decision to pursue revascularization in stting of peripheral vascular disease is
NOT dependent on ABI cut-off and instead is indicated in setting of (1) Rest pain (2)
Ischemic ulceration (arterial ulcer) (3) Gangrene
Claudication: Patient will have risk factors for peripheral vascular disease (diabetes, HTN,
smoking, etc.), reduced lower extremity pulses, reduced lower extrmity temperature, pain
classically in the calves, reduced hair on legs
Pseudoclaudication (spinal stenosis): Positional (improves with exion), classically
affects buttocks and thighs, may be associated with back pain

29. A 67-year-old man comes to the physician because of a …

Thymoma

Presentation is most consistent with myasthenia gravis (tends to affect young women
and older men and classically leads to dysphagia, dysarthria and eye weakness that is
worse at the end of the day)
Key idea: Patients with myasthenia gravis should be treated with an acetylcholinesterase
inhibitor (most often pyridostigmine), but they should also have a CT scan performed
because in about 15% of cases the cause is a thymoma and some patients will be cured
following removal of the thymoma
Note: If patient with weakness is described in vignette and they give you some sort of
chest imaging, you should have high index of suspicion for myasthenia gravis
(associated with thymoma) or Lambert-Eaton (associated with small cell lung cancer)
https://www.pennmedicine.org/for-health-care-professionals/for-physicians/physician-education-and-
resources/clinical-brie ngs/2019/march/medical-and-surgical-management-of-myasthenia-gravis-and-
thymoma

30. A 42-year-old woman comes to the physician because of right…

Endoscopic retrograde cholangiopancreatography

Patient who had gallbladder removed 2 months ago presents with right-sided abdominal
pain, clinical jaundice, cholestatic pattern of LFTs (increased direct bilirubin and increased
alkaline phosphatase) along with dilated intrahepatic biliary ducts, most consistent with a
retained stone within the common bile duct that needs to be investigated and treated by
ERCP
Key idea: Although patient had gallbladder removed, patients can sometimes have
residual stones within the gallbladder remnant, cystic duct or common bile duct that
remain behind and can cause pathology
Key idea: ERCP indicated in setting of choledocolithiasis because of its association with
acute cholangitis and the morbidity/mortality associated with that disease
Note: HIDA scan (cholescintigraphy) is often not the answer on NBME exams, with its
main indication being in a patient with suspected acute cholecystitis who had negative
ndings on RUQ ultrasound

31. A 37-year-old woman, gravida 2, para 1, at 12 weeks’ gestation…


Previous preterm delivery

The two most important risk factors for preterm labor include (1) History of previous
preterm labor (2) Short cervical length (<2 cm in patient without history of preterm
labor or <2.5 cm in patient with history of preterm labor)

32. A previously healthy 16-year-old high school wrestler comes…

Eczema herpeticum

Patient with baseline atopic dermatitis (atopic patient [allergic rhinitis] with erythema and
licheni cation over antecubital and popliteal fossae) who presents with acute painful,
umbilicated vesicles over sites of atopic dermatitis is most consistent with eczema
herpeticum caused by HSV-1
Note: Patient is a wrestler, which in NBME language means they have high risk of being
exposed to skin infection because they are rolling around on sweaty mats
Key idea: Four infectious complications of atopic dermatitis include (1) Eczema
herpeticum [HSV-1, painful vesicular rash] (2) Impetigo (Staph or Strep, painful pustules
with honey-colored crust) (3) Molluscum contagiosum (Poxvirus, umbilicated esh-
colored papules) (4) Tinea corporis (Tricophyton, pruritic patch with central clearing and
raised border with overlying scale)

33. A 42-year-old woman comes to the physician for evaluation of persistently…

Autonomous production of aldosterone

Key idea: Young, otherwise healthy women should not present with severe essential
hypertension, and it should make you have a high index of suspicion for secondary
hypertension ( bromuscular dysplasia, hyperaldosteronism, coarctation of the aorta,
Cushing’s syndrome, hypo/hyperthyroidism, pheochromocytoma, etc.)
In this patient, her labs demonstrate a hypokalemic metabolic alkalosis, which is
consistent with a state of elevated RAAS activity either secondary to poor renal perfusion
(question would tell you a bruit is present) or endogenous, inappropriate production of
aldosterone

34. A 67-year-old man has had shortness of breath on exertion…

Thoracentesis

Older patient with signi cant smoking history and subacute weight loss who appears
chronically ill presents with a large left-sided pleural effusion and should therefore receive
thoracentesis in order to determine the etiology of the effusion (which is most likely
malignant)
Note: Only reason to not get a thoracentesis in the setting of a new pleural effusion is if
you have very high index of suspicion for heart failure and the patient’s effusion responds
well to diuretic therapy
Key idea: Pleural effusion with malignant cells seen on cytology in setting of lung cancer
consistent with Stage 4 disease because it is caused by metastasis of lung cancer to
the pleura and not by pleural uid production triggered by the primary lung cancer
Key idea: When trying to decide where to sample/biopsy in setting of suspected cancer,
one of the considerations (in addition to risk of procedure and access to site) includes
trying to select the site that would enable the most up-staging of the cancer because it
has important prognostic and therapeutic implications; in this case, if the pleural uid is
found to have malignant lung cancer cells, then the patient has automatic Stage 4 disease

35. A 26-year-old woman is brought to the emergency department…

ACTH stimulation test

Young female patient with autoimmune history presents with fatigue, weakness,
abdominal complaints and generalized hyperpigmentation found to have eosinophilia,
hyponatremia, hyperkalemia and metabolic acidosis on labs, most consistent with
Addison’s disease which should be worked up with an ACTH stimulation test (would
typically lead to increased cortisol levels in healthy patient but will lead to a less robust
response in patient with Addison’s disease)
Key idea: High cortisol/steroid states lead to increased neutrophil count, decreased
lymphocyte count and decreased eosinophil count, so patients with Addison’s disease
have the opposite
Key idea: Addison’s disease leads to primary failure of adrenal gland, such that the
aldosterone-producing cells are also affected, leading to the hyperkalemia, hyponatremia
and metabolic acidosis because aldosterone typically works at collecting tubule to
reabsorb sodium in exchange for potassium and protons
Key idea: Hyperpigmentation (in addition to hyperkalemia) helps to distinguish primary
from secondary/tertiary adrenal failure because in primary adrenal failure (Addison’s) the
pathology is in cortisol production by the adrenal, so there will be less negative feedback
of cortisol on the pituitary, leading to increased production of ACTH, with increased
production of ACTH also leading to increased production of melanocortin because they
share the same precursor protein, with melanocortin leading to hyperpigmentation
[contrast with secondary/tertiarty adrenal failure where ACTH is low]

36. A 40-year-old man is brought to the emergency department…


Myocardial contusion

Patient involved in a motor vehicle accident presents with severe chest pain, hypotension
and arrhythmia and decompensates further with administration of uids (because patient
is basically in cardiogenic shock and cannot handle increased volume), most consistent
with myocardial contusion in setting of blunt cardiac injury
Myocardial infarction would lead to ST-segment changes, pulmonary contusion would
primarily lead to respiratory symptoms and traumatic rupture of the aorta would lead to
severe hemodynamic instability and likely a positive response to uids (although most of
these patients die in the eld due to the high volume of blood loss they experience)

37. A 32-year old man with alcoholism is brought to the emergency…

Vitamin B1 (thiamine)

Patient with chronic alcoholism present with confusion, ataxia and ophthalmoplegia, most
consistent with Wernicke encephalopathy secondary to thiamine de ciency
Key idea: On the NBME exam, patient with alcoholism = patient with malnutrition and
vitamin de ciencies (among other associations)

38. A 72-year-old man with hypertension has had increasingly severe…

Metastatic prostate carcinoma

Older man with back pain, tenderness over spine, and osteoblastic vertebral lesions most
consistent with metastatic prostate cancer
Mnemonic for causes of osteoblastic bone metastases: Holster, Point, Shoot and Blast
the enemy – Hodgkin lymphoma, Prostate cancer, Small cell lung cancer lead go
osteoblastic lesions

39. A 6-month-old girl is brought to the physician for a …

Obstruction of cerebrospinal uid ow

Question stem tells us that patient has increased intracranial pressure secondary to
hydrocephalus (increasing head circumference with bulging fontanelles)
Epidemiologically the most common cause of hydrocephalus in an infant is aqueductal
stenosis (obstruction of CSF ow from the 3rd ventricle to the 4th ventricle) and this
patient may have additional CSF ow obstruction secondary to an Arnold Chiari type 2
malformation (associated with myelomeningocele)
Arnold-Chiari type 1 malformation associated with syringomyelia, whereas Arnold-Chiari
type 2 malformation associated with myelomeningocele (with this patient has)

40. A 47-year old man is admitted to the hospital after threatening…

Haloperidol decanoate

Patient has a presentation consistent with schizophrenia that is responsive to


antipsychotic treatment, but with multiple relapses due to poor medication adherence
Patients with unstable psychiatric illness who have frequent medication non-adherance,
poor insight or little social support are good candidates for long-acting injectable
antipsychotic, with one of the common formulations being haloperidol decanoate
Contrast with clozapine, which is indicated in setting of treatment-resistant
schizophrenia and schizophrenia with suicidality

41. A newborn is in severe respiratory distress immediately …

Mother: O, Rh-negative / Newborn: O, Rh-positive

Newborn born to mother without prenatal care who has previously been pregnant (G2P1)
is found to have anasarca (diffuse body edema) with signi cant edema and positive direct
Coombs’ test, most consistent with Rh hemolytic disease
Key idea: Anasarca/hydrops fetalis occurs due to high output heart failure secondary to
profound anemia seen in Rh hemolytic disease
Key idea: ABO incompatibility will not present with as severe of a presentation because
majority of maternal antibodies against other AABO blood groups are IgM and do not
readily cross the placenta (contrast with acquired Rh-targeting antibodies in Rh-negative
mother who has been exposed to Rh-positive blood which are IgG and can easily cross the
placenta)
Potential causes of anasarca or hydrops fetalis: Rh-hemolytic disease, parvovirus/CMV
infection of mother, hemoglobin Barts disease (severe form of alpha thalassemia), etc.

42. A 19-year-old man is brought to the emergency department 45…

Laparotomy

Indications for urgent exploratory laparotomy (ex-lap) in setting of penetrating abdominal


trauma includes (1) Hemodynamic instability (which this patient has) (2) Peritonitis
(rebound, guarding) (3) Evisceration (externally exposed organs) (4) Blood from NG
tube or on rectal exam
Note: If patient has no indications for immediate ex-lap, then next step would be local
exploration of the wound and an extended FAST scan

43. A 52-year-old man comes to the physician for a routine health…

Exercise stress test

2 key indications for an exercise stress test include (1) History compatible with coronary
artery disease (angina, etc.) (2) Risk strati cation prior to starting exercise plan
Spirometry would be used to evaluated dyspnea and cardiac catheterization would only
be used if patient had positive exercise stress test

44. A 32-year-old woman who is HIV positive has a CD4+…

All patients (HIV or not) should receive the annual In uenza vaccine and the Tdap every 10
years
Additional vaccination requirements for patients with HIV include (1) Vaccination for
hepatitis B unless they have documented immunity (2) Strep Pneumo PCV13 followed
by the 23-valent PPSV23 8 weeks later and again in 5 years and at age 65 (3)
Meningococcal vaccine with boosters every 5 years

45. A 16-year-old girl is brought to the physician because of episodes…

Aortic incompetence

Patient with Marfanoid habitus (tall, long arms, long ngers) presents with heart
symptoms and is found to have bounding peripheral pulses (consistent with widened
pulse pressure) and an early diastolic murmur without clear localization most consistent
with aortic regurgitationt/incompetence
Key idea: Patient’s with Marfan’s syndrome are prone to aortic aneurysm, with a thoracic
aneurysm leading to widening of the aortic valve ring with resulting aortic incompetence
Note: Localization of aortic regurgitation murmur can sometimes help to detemine the
location of the pathology, with loudest murmur at right sternal border most consistent
with aortic root disease and loudest murmur at left sternal border most consistent with
primary valvular disease

46. A 3-year-old boy is brought for a follow-up examination…

CT scan of the head


Presentation most consistent with mastoiditis (patient with acute otitis media who
develops pain behind the ear with displacement of the auricle
Patients with mastoiditis should be worked-up with CT scan of the head in order to look
for potential complications such as abscess that would affect management
Patients with pure mastoiditis without abscess: IV antibiotics
Patients with mastoiditis and abscess: IV antibiotics +/- tympanostomy or mastoidectomy

Exam section 4:

1. A study is conducted to compare the effectiveness of lorazepam…

Confounding variables

Patients in the lorazepam group were much more likely to receive a paralytic agent, which
is an important confounder and calls into question the internal validity of the study
Key idea: Randomization is often performed in studies with the idea that it swill equally
disperse potential confounders between the control and treatment arms of the study

2. A 70-year-old man comes to the physician because of…

Out ow obstruction of the bladder

Patient with chronic urinary hesitancy and frequency who is afebrile is found to have an
enlarged nontender prostate and a positive urinalysis (pyuria and bacteria), most
consistent with asymptomatic bacteriuria in the setting of BPH
Key idea: For men, BPH with bladder outlet obstruction is reported to be the major
predisposing factor for the development of asymptomatic bacteriuria (positive urinalysis
but asymptomatic) because it leads to urine stasis
Infection of epididymis would lead to unilateral posterior testicular pain improved with
testicular elevation, infection of urethra would lead to UTI symptoms (dysuria, suprapubic
tenderness), and acute prostatitis would lead to systemic symptoms (fever, chills,
malaise)and tender prostate on DRE

3. A 15-year-old boy is brought to the emergency department…

Hypertrophic obstructive cardiomyopathy

Presentation of a young healthy child with exertional loss of consciousness without a


prodrome found to have a systolic ejection murmur in the aortic/pulmonic valve
distribution that becomes louder with decreased preload (upon standing) most consistent
with HOCM
Key idea: While vasovagal syncope would be most common cause of LOC, it is preceded
by prodrome of nausea, sweating, etc.
Key idea: Most murmurs improve with decreases in preload (upon standing), with the two
exceptions being HOCM (decreased preload leads to less stretching of the LV and
therefore more obstruction near aortic valve due to septal hypertrophy) and Mitral valve
prolapse (less preload leads to less stretching of the LV, leading to less taut chordae
tendinae which will be more prone to relaxation and enable earlier prolapse of the mitral
valve lea et)

4. A 62-year-old woman is brought to the emergency department 1 hour…

Orthostatic hypotension

Older patient recently started on anticoagulation therapy who has been passing dark
bloody stools for 2 days presents with loss of consciousness upon standing found to have
hypotension and tachycardia on exam, most consistent with orthostatic hypotension
Key idea: Orthostatic hypotension formally diagnosed if patient has drop in blood
pressure by at least 20 systolic and/or at least 10 diastolic upon standing OR if patient has
consistent light-headedness or LOC upon standing

5. A previously healthy 52-year-old woman comes to the physician…

Sporotrichosis

Sporothrix schenckii is a dimorphic fungus found in decaying plant matter and soil with
can often lead to an ulcerative skin papule with proximal lesions along lymphatic chain in
patients with exposure to plants (often in gardeners)

6. A 57-year-old man comes to the physician because of intermittent…

Over ow incontinence from acontractile bladder

Patient with history of poorly controlled T2DM with signs of autonomic neuropathy
(gastroparesis) presents with intermittent urinary incontinence found to have a post-void
residual volume of 500 mL most consistent with over ow incontinence secondary to
poor bladder contraction
Key idea: Post-void residual > 150 mL is a sign of urinary retention
Key idea: Autonomic neuropathy can be seen in late-stage diabetes and can manifest as
problems with esophageal motility (dysphagia), gastric emptying (gastroparesis),
intestinal function (constipation), blood pressure (orthostatic blood pressure
unresponsive to uids) and bladder function (acontractile bladder, incontinence)
7. A 47-year-old woman comes to the physician for a follow-up…

Gastric bypass

Patient is a middle-aged woman with a BMI of 75, poorly controlled type 2 diabetes and
signi cant limitations in activities of daily living due to obesity who is compliant with her
diet and medications and has been unsuccessful at losing weight on her own who would
be a candidate for gastric bypass
Indications for gastric bypass: Patient with multiple failed attempts to lose weight with
adequate ability to follow-up and adhere to a plan who also has (1) BMI > 40 or (2) BMI
>35 with obesity-related comorbidity (Type 2 diabetes, obstructive sleep apnea,
obesity hypoventilation syndrome, debilitating osteoarthritis, cardiomyopathy,
coronary artery disease)
Note: Interestingly, bariatric surgery in patients with comorbid type 2 diabetes has been
found to signi cantly improve and even cure type 2 diabetes in a signi cant proportion of
patients (https://care.diabetesjournals.org/content/34/Supplement_2/S361)

8. A 16-year-old boy with neuro bromatosis is brought for a …

Catecholamine-producing tumor

Young patient with neuro bromatosis who presents with episodes of headaches and
ushing who is found to have elevated blood pressure, most consistent with
pheochromocytoma
Neuro bromatosis Type 1: Cafe-au-lait spots, intellectual disability, cutaneous
neuro bromas, Lisch nodules, optic glioma, Pheochromocytomas, seizures (often
secondary to meningioma)
Neuro bromatosis type 2: Leads to bilateral (2) vestibular schawannomas, affects both
(2) eyes (juvenile cataracts and leads to 2 other types of brain tumor (meningioma,
ependymoma

9. A 17-year-old boy sustains a head injury and loses consciousness…

Acute respiratory distress syndrome

Patients who have a near-drowning event are still at risk for morbidity/mortality from
delayed or secondary drowning hours to days later
The underlying pathophysiology is related to in ammation secondary to the aspiration of
water into the lung, and the way that I remember the association between near-drowning
and ARDS is that the inhaled water washes away the surfactant leading to ARDS,
analogous to how preterm newborns can have neonatal respiratory distress syndrome
due to low surfactant levels

10. Four days after undergoing resection of an obstructing sigmoid…

Central venous catheter

Most common types of hospital-acquired infections are central-line associated


bloodstream infections (CLABSI) which often lead to Staph infection, Catheter-
associated Urinary tract infections (CAUTI) which often lead to E. coli infection, Surgical
Site Infection which often leads to Staph or Strep and Ventilator-associated pneumonia
which leads to Strep pneumo, H. in uenzae, Staph, etc. infections
Intra-abdominal abscess would most commonly lead to anaerobic, gram-negative
infection and while operative wound could lead to Staph or Strep infection you would
expect the colostomy to be erythematous, purulent, etc.

11. A 28-month-old boy has a history of cyanosis since birth with…

Tetralogy of Fallot

Infant/toddler with early cyanosis and clubbing with murmur consistent with RV
hypertrophy (RV heave), Pulmonic stenosis (single S2), and a VSD (systolic murmur),
which are 3/4 of the features of Tetralogy of Fallot
Key idea: Causes of early cyanosis from primary heart defects can be remembered with
mnemonic 1, 2, 3, 4 and 5 because it can be caused by (1) persistent truncus arteriosus
(1 vessel) (2) transposition of the great vessels (2 switched vessels) (3) Tricuspid atresia
(Tri = 3) (4) Tetralogy of Fallot (Tetra = 4) (5) Total anomalous pulmonary venous return
(5 letters in name)

12. Over the past 3 months, a 30-year-old woman has had intermittent…

Adrenal gland

Young otherwise healthy woman with episodes of the 5 P’s: elevated blood Pressure,
Painful headache, Perspiration, Palpitations and Pallor

13. A 57-year-old woman is brought to the physician 2 days…

Osteoporotic compression fracture


Middle-aged woman with risk factor for vertebral compression fracture (long-term
prednisone use) presents with sudden onset back pain and tenderness to percussion over
the spine, most consistent with a vertebral compression fracture
Key idea: Vertebral compression fracture comes in 2 avors (1) Chronic fracture:
Painless with progressive kyphosis and loss of stature (2) Acute fracture: Low back pain
with decreased spinal mobility + tenderness at affected level
Key idea: Causes of point tenderness over vertebral body include compression vertebral
fracture, osteomyelitis and metastatic disease to vertebral bone

14. A previously healthy 32-year-old plumber comes to the…

Muscle strain

Young man who developed back pain after lifting heavy equipment and who has
paraspinal tenderness (muscles run along either side of spine) with negative straight-
leg testing and no neurological symptoms, most consistent with muscle strain (most
common cause of back pain in young otherwise healthy people)

15. A 57-year-old man comes to the physician because of a 1-day history…

Indomethacin

Middle-aged man who presents with atraumatic right knee arthritis found to have cloudy
synovial uid with a WBC count of 9000 (normal synovial uid has <200 WBCs) and
negatively birefringent, needle-shaped crystals which are classic for acute gout
Key idea: Acute gout should be treated with NSAIDs (often indomethacin) and patients
should NOT be started on chronic gout drugs (allopurinol, probenecid, etc.) until the acute
are has resolved because these drugs can lead to rapid shifts in uric acid levels that can
exacerbate/cause a new are
Note: Patients can sometimes be given oral or intra-articular steroids, but NSAIDs are
tried rst due to lower risk and good effectiveness in the majority of patients

16. A 67-year-old woman has been intubated for 1 week…

Wean from the ventilator

3 major criteria for extubation of patient includes (1) pH > 7.25 (not retaining CO2) (2)
Adequte oxygenation on minimal support (FiO2 40% or less and PEEP 5 mm Hg or less)
(3) Intact inspiratory effort and suf cient mental alertness to protect the airway
Key idea: As with all other interventions/tubes (Foley, central line, etc.), we only want to
use them as long as necessary because they are not harmless interventions
Key idea: FiO2 and PEEP are responsible for controlling oxygenation while on ventilator,
whereas tidal volume and respiratory rate (which when multiplied equals minute
ventilation) are responsible for ventilation (CO2 levels)

17. A 42-year-old man is brought to the physician…

Complex partial seizures

Although terminology has fallen out of favor in some neurology spheres, simple seizures
lead to no loss of consciousness and complex seizures lead to loss of consciousness (like
this patient who has staring spells where he smacks his lips and does not respond)
Patient has a partial seizure rather than a generalized seizure because he does not show
tonic-clonic movements and his aura of smelling burnt rubber and hearing an intense
hissing localizes to the temporal lobe, which is the most common cause/site of partial
seizures
Important to contrast with absence seizures, which often last for less than 20 seconds
(in contrast to 30-90 seconds of partial seizures) and are accompanied by simple
automatisms (eyelid uttering, lip smacking)

18. A 20-year-old African American man with sickle cell…

Increased demand for folic acid

Young man with sickle cell anemia who presents with a macrocytic anemia with an
abnormally low reticulocyte count, most likely due to folate de ciency
Key idea: Folate de ciency often related to (1) Chronic hemolysis (such as in sickle cell
disease) (2) Poor dietary intake (such as in alcoholic) (3) Malabsorption (such as in
gastric bypass or tropical sprue) and (4) medications (methotrexate, phenytoin, etc.)

19. A 42-year-old man comes to the physician because of a …

Emphysema

Young man with family history of “lung and liver problems” and 25 pack-year smoking
history presenting with chronic progressive shortness of breath with expiratory wheezes
found to have elevated LFTs on exam, most consistent with emphysema in the setting of
alpha-1 antitrypsin syndrome
Key idea: Alpha-1 antitrypsin is a co-dominant disorder, with homozygous patients
developing emphysema and cirrhosis and heterozygous patients being highly prone to
emphysema with smoking
20. A healthy 24-year-old woman comes for a routine health…

Repeat examination in 2 weeks

Young otherwise healthy woman with no family history of ovarian cancer who presents
with a unilateral tender adnexal mass while in the luteal phase of her menstrual cycle,
most consistent with a functional ovarian cyst
Key idea: Best next step would be repeating the exam during the proliferative phase of
the menstrual cycle, where we would expect the mass to go away or improve after
ovulation has taken place and the egg has been released
Key idea: Patients with a persistent mass or a similar presentation in an older patient
should have full work-up performed

21. A previously healthy 87-year-old woman comes to the physician…

Lichen sclerosus

Description is most compatible with lichen sclerosus because the vulvar thinning is
diffuse (rather than discrete area of pathology seen in patient with vulvar carcinoma)

22. A 37-year-old primigravid woman at 9 weeks’ gestation…

Antiemetic therapy

Although it is normal for women to have nausea/vomiting during the rst trimester, when
they are having severe, persistent vomiting with signs of dehydration and orthostasis,
>5% loss of pre-pregnancy weight, or lab ndings of ketonuria (pathognomonic of
hyperemesis gravidarum > normal pregnancy nausea) then they have hyperemesis
gravidarum
Patients with hyperemesis gravidarum should be admitted to the hospital and treated
with antiemetics and intravenous uids

23. A 62-year-old woman is brought to the physician by her…

Nitroprusside

Patient with blood pressure > 180/20 and papilledema and encephalopathy (confusion,
headache, etc.) most consistent with hypertensive encephalopathy
Key idea: Potential treatments of hypertensive emergency/encephalopathy include (1)
Nitroprusside (2) Labetalol (3) Fenoldopam (4) Clevidipine, nicardipine
24. A previously healthy 47-year-old woman comes to the…

Gallstone pancreatitis

Middle-aged woman with risk factors for gallstones (Female, Forty, Fertile) and history
consistent with biliary colic who presents with nausea, vomiting, epigastric and RUQ
tenderness and lab ndings of elevated lipase and amylase most consistent with
gallstone pancreatitis
Key idea: 2 main causes of pancreatitis are gallstones and alcohol, with elevated
triglycerides leading to pancreatitis only when triglyceride levels are >1000-2000
Key idea: In setting of gallstone pancreatitis, important to make sure patient does not
have concomitant ascending cholangitis (fever, RUQ pain, jaundice, hypotension, altered
mental status) because then patient would require urgent ERCP + antibiotics

25. An 18-year-old man comes to the physician 1 week after he…

Operative treatment

Young otherwise healthy patient with hypertension should prompt high index of suspicion
for a secondary cause of hypertension ( bromuscular dysplasia, hyperaldosteronism,
coarctation of the aorta, Cushing’s syndrome, hypo/hyperthyroidism,
pheochromocytoma)
In this patient who has increased muscular build and stronger pulses in the upper
extremities and signs of LV hypertrophy on ECG (secondary to chronically increased
afterload), the most likely diagnosis is coarctation of the aorta which requires operative
treatment

26. A 6-year-old boy is brought to the physician by his mother…

An excess of very long chain fatty acids

Child with a genetic X-linked disorder (maternal uncle had similar symptoms) found to
have a variety of neural symptoms and diffuse white matter disease on MRI, consistent
with Adrenoleukodystrophy (rare peroxisomal disorder of beta-ooxidation that leads to
VLCFA buildup in adrenal glands, white matter of brain and testes)

27. A 30-year-old woman comes to the physician for a …

Serum thyroid-stimulating hormone concentration


Patient with bipolar disorder who was started on lithium and has experienced weight gain
most consistent with a picture of lithium-induced hypothyroidism
Key idea: Patients on lithium should have regular TSH monitoring every 6-12 months
regardless of symptoms
Key idea: Lithium is associated with hypothyroidism, nephrogenic diabetes insipidus,
chronic kidney disease and hyperparathyroidism

28. An 80-year-old woman has had bleeding gums for 3 weeks…

Elderly woman with bland diet with bleeding gums, ecchymoses and perifollicular
hemorrhages, which are the classic symptoms seen in scurvy (vitamin C de ciency)
Pathophysiology involves problem in collagen synthesis because vitamin C involved in
hydroxylation of proline and lysine

29. A 45-year-old woman comes to the emergency department…

Lorazepam

Patient’s presentation is consistent with a panic attack (leads to combination of


paresthesias, palpitations, abdominal pain, nausea, intensee fear of dying/losing control,
light-headedness, chest pain, choking, sweating, shaking and shortness of breath), with
panic disorder being acutely treated with benzo’s (such as lorazepam) and
prophylactically/chronically treated with SSRIs
Note: Patients often develop tingling around the lips secondary to hypocalcemia which
occurs because patients begin to breathe rapidly and deeply, leading to a respiratory
alkalosis, which leads to more calcium binding to negatively-charged albumin, leading to a
drop in free calcium

30. A healthy 23-year-old woman, gravida 1, para 1…

Oral levonorgestrel now and again in 12 hours

Four options for emergency contraception include (1) Copper IUD (most effective) (2)
Ulipristal (3) Levonorgestrel (“Plan B”) (4) Oral contraceptives

31. A 5-year-old girl is brought to the physician because…

Empyema
Child who has been incompletely treated for pneumococcal pneumonia who presents
with high fever, nonproductive cough and diminished breath sounds over lower lung elds
with dullness to percussion (suggestive of some form of pleural effusion), most consistent
with an empyema
Bronchopleural stula often seen after trauma, lung abscess often seen secondary to
aspiration, pleurodynia leads to sudden sharp chest pain and pneumothorax would lead to
diminished breath sounds with hyperresonance to percussion

32. A 67-year-old woman is brought to the emergency department…

Esophageal perforation

Older woman who received an upper endoscopy with associated procedure presents with
severe chest pain, hematemesis, crepitus in the neck and positive blood in the GI tract
most consistent with an esophageal perforation
Key idea: Best diagnostic test would be esophagography with water-soluble contrast and
patient should be treated with IV antibiotics, PPIs and emergency surgery
Contrast with Mallory-Weiss tear, which is a partial tear and therefore would not lead to
crepitus and would often lead to more profound hematemesis

34. A 22-year old primigravid woman at 16 weeks’ gestation is…

Choriocarcinoma

Young pregnant woman who presents with enlarged uterus (uterus to umbilicus
consistent with gestational age of 20+ weeks), absent fetal heart tones, severely elevated
beta0hCG and chest x-ray showing multiple densities (“cannonball metastases”) most
consistent with choriocarcinoma
Key idea: Rare diagnosis that can develop during pregnancy, after delivery or after
abortion
Key idea: Can be associated with symptoms related to increased hCG due to alpha
subunit of hCG being the same as the alpha subunit of LH, FSH and TSH, including early
pre-eclampsia, hyperemesis gravidarum, hyperthyroidism and theca-lutein cysts

35. An asymptomatic 21-year-old woman is found to have an…

Benign cystic teratoma

Image demonstrates serosal lining, hair and a tooth/bone and therefore is consistent with
a teratoma (tumor made up of several different types of tissue)
Key idea: Most common cause of an adnexal mass in a young woman is a teratoma, and it
can be associated with ovarian torsion

36. A 13-year-old boy is brought to the physician by his mother…

No pharmacotherapy is indicated

Children with hypertension (BP > 95th percentile) are primarily treated with lifestyle
changes (weight loss, low-sodium diet, regular physical activity)
Indications for treatment of hypertension in children: (1) Symptomatic hypertension
(headaches, etc.) (2) secondary hypertension (3) end-organ damage (4) diabetes (5)
hypertension refractory to lifestyle changes
Key idea: In both children and adults, lifestyle interventions should be thoroughly tried
before getting medications on board in setting of hypertension

37. A 72-year-old man comes to the physician because of a …

Oral enalapril therapy

Elderly man with chronic diabetes found to have signs of CKD (elevated BUN and
creatinine) and albuminuria who could have bene tted from an ACE inhibitor or ARB
Key idea: Two most common causes of CKD include hypertension and diabetes, with
diabetes leading to nephropathy because it leads to preferential arteriosclerosis of the
efferent arteriole > afferent arteriole, thus leading to high pressure in glomerulus,
resulting elevated GFR and damage to glomerulus over time (ACE inhibitor or ARB can
reduce this damage by dilating the afferent arteriole, leading to a decreased GFR)
Key idea: ACE inhibitors or ARBs should be initiated at onset of microalbuminuria in all
diabetic patients

38. A 57-year-old man is brought to the physician by his wife…

Withold the results as the patient wishes

Patient’s have the right to decline when offered information about their health or lab
results

39. A previously healthy 3-month-old girl is brought to…

Dysmorphogenesis of the third and fourth pharyngeal pouches


Infant presenting with grunting and breathing dif culty found to be hypocalcemic with
absent thymic shadow and infection with an organism commonly seen in patients with
HIV and other forms of cellular (T cell) de ciency, most consistent with DiGeorge
syndrome
Mnemonic for symptoms for CATCH-22: Cardiac abnormalities (often Tetralogy of Fallot),
Abnormal facies, Thymic aplasia, Cleft palate/lip, Hypocalcemia (secondary to parathyroid
a/dysgenesis)

40. A 67-year-old man is brought to the emergency department…

Brain abscess

Elderly patient with a history of aortic valve replacement presents with fever, new systolic
murmur and signs of septic emboli to the brain, most consistent with infective
endocarditis
Although brain abscess is a rare complication of endocarditis (1-7%), it is a feared
complication of septic emboli to the brain and is more likely to occur than the other
options listed

41. A 32-year-old man comes to the physician because of…

Vitamin D de ciency

Young patient with malabsorption (intermittent diarrhea with weight loss and fat in the
stool) who has low calcium and phosphorous levels, which is most consistent with vitamin
D de ciency
Key idea: Vitamin D responsible for calcium and phosphorous absorption from the GI tract
(contrast with PTH which leads to increased calcium and decreased phosphate)
Key idea: Fat soluble vitamins are vitamins A, D, E and K

42. A 4-week-old newborn is brought to the physician because…

High pulmonary vascular resistance

Newborn presents with labored breathing with cardiomegaly, increased pulmonary


vascular markings, RVH on ECG and a holosystolic murmur at the lower left sternal border
that was not present at birth, which is most consistent with a ventricular septal defect
Key idea: In utero the pulmonary vascular resistance is very high because the lungs are
not participating in gas exchange and as the baby acclimates to the outside world the
pulmonary vascular resistance steadily drops; because of this, left-to-right murmurs can
increase in severity over the rst several weeks of life because blood meets less
resistance moving from the LV to the RV to the pumonary arteries
Key idea: Blood (like most things in this world) will follow the path of least resistance!

43. A 32-year-old woman comes to the physician because of bright…

Anesthetic ointment and stool softeners

Young healthy woman who has bloody stools with severe pain and an anal ssure on
physical exam, most consistent with an anal ssure
Key idea: Most anal ssures can be managed conservatively with anesthetic ointment
and stool softeners (highly associated with mild constipation, as is the case with our
patient)

44. A 67-year-old woman comes for a routine health maintenance…

Measurement of serum parathyroid hormone conentration

Healthy patient with increased calcium and decreased phosphorous, most consistent with
elevated PTH levels with the most common cause being primary hyperparathyroidism
Key idea: Parathyroid hormone leads to increased calcium and decreased phosphorous,
whereas vitamin D leads to increased calcium and increased phosphorous
Key idea: If patient had a positive protein gap (total protein – albumin > 4 g/dL), then you
would have higher index of suspicion for multiple myeloma and would perform a serum
protein electrophoresis

45. One month after undergoing an uneventful renal…

Increased dosage of corticosteroids

Patient with renal transplant who develops increased BUN and creatinine on order of
weeks to months most concerning for acute transplant rejection (which is con rmed by
biopsy in this patient)
Key idea: First-line treatment for acute rejection are steroid boluses vs. antilymphocyte
agents vs. antithymocyte serum

46. A 72-year-old man comes to the physician with his wife…

Measurement of blood lead concentration


Patient with risk factor for lead poisoning (homemade whiskey) presents with abdominal
pain, neurologic symptoms, and a microcytic anemia, which are the three classic
symptoms of lead poisoning
Key idea: Potential buzzwords that should trigger thought of lead poisoning include
battery factory, homemade whiskey, renovating old house, etc.
Note: While not a high-yield association, lead poisoning can lead to worsening of gout

We are not af liated with the NBME, USMLE or AAMC.

The answer explanations may not be reproduced or distributed, in whole or in part, without written permission of Step

Prep.

Share this:

 Twitter  Facebook

Related

Step 2 CK (CCSSA) Form 8 Step 2 CK (CCSSA) Form 7 NBME Step 2 Free 120
March 8, 2020 March 8, 2020 June 27, 2020
In "Step 2 CK (CCSSA) Form In "Step 2 CK (CCSSA) Form Similar post
8" 7"

S T E P 2 C K ( C C S S A ) FO R M 6

C C S S A FO R M 6 , S T E P 2 C K FO R M 6

You might also like